Last visit was: 23 Apr 2024, 13:07 It is currently 23 Apr 2024, 13:07

Close
GMAT Club Daily Prep
Thank you for using the timer - this advanced tool can estimate your performance and suggest more practice questions. We have subscribed you to Daily Prep Questions via email.

Customized
for You

we will pick new questions that match your level based on your Timer History

Track
Your Progress

every week, we’ll send you an estimated GMAT score based on your performance

Practice
Pays

we will pick new questions that match your level based on your Timer History
Not interested in getting valuable practice questions and articles delivered to your email? No problem, unsubscribe here.
Close
Request Expert Reply
Confirm Cancel
SORT BY:
Date
Tags:
Show Tags
Hide Tags
Current Student
Joined: 31 Jul 2017
Status:He came. He saw. He conquered. -- Going to Business School -- Corruptus in Extremis
Posts: 1734
Own Kudos [?]: 5734 [20]
Given Kudos: 3054
Location: United States (MA)
Concentration: Finance, Economics
Send PM
Most Helpful Reply
Current Student
Joined: 31 Jul 2017
Status:He came. He saw. He conquered. -- Going to Business School -- Corruptus in Extremis
Posts: 1734
Own Kudos [?]: 5734 [6]
Given Kudos: 3054
Location: United States (MA)
Concentration: Finance, Economics
Send PM
General Discussion
Intern
Intern
Joined: 08 Feb 2019
Posts: 8
Own Kudos [?]: 10 [0]
Given Kudos: 11
Location: India
Concentration: Finance, International Business
GPA: 3.8
Send PM
Manager
Manager
Joined: 19 Apr 2017
Posts: 84
Own Kudos [?]: 133 [0]
Given Kudos: 40
Concentration: General Management, Sustainability
Schools: ESSEC '22
GPA: 3.9
WE:Operations (Hospitality and Tourism)
Send PM
Re: Newspaper subscriber: Arnot's editorial argues that by making certain [#permalink]
Should it not be trust(/-ed)? (typo)

What a question i spent 5 mins

Part 1(Arnot's argument): X change would solve Y
Part 2 (Newspapers Conclusion): claim X is dependent on assumption P and therefore false.

After reading answer choice D and E i felt like the question was asking one to find the role of part 2 (do correct me if I'm wrong? cause when i tried to find weakener i was stuck between a and c)

A says it rejects X because part 1 provides insufficient evidence and is therefore false (part 2 is doing just that)

B merely paraphrases part 1

C says the assumption may be true but the conclusion is false. but Part 2 says that assumption is false and hence conclusion
is false )

D Part 2 does not distort the argument. Part 2 says it just depends on a doubtful assumption

E does not specify role of Part 2. even if the term is used interchangeably.
Director
Director
Joined: 03 Mar 2017
Posts: 586
Own Kudos [?]: 418 [0]
Given Kudos: 596
Location: India
Concentration: Operations, Technology
Send PM
Re: Newspaper subscriber: Arnot's editorial argues that by making certain [#permalink]
GMATNinja generis VeritasKarishma

Please help. What is wrong with option C??

I somehow feel that this is not the right question to practice for GMAT. Am I correct in saying this??
Tutor
Joined: 16 Oct 2010
Posts: 14816
Own Kudos [?]: 64880 [2]
Given Kudos: 426
Location: Pune, India
Send PM
Re: Newspaper subscriber: Arnot's editorial argues that by making certain [#permalink]
2
Kudos
Expert Reply
warrior1991 wrote:
GMATNinja generis VeritasKarishma

Please help. What is wrong with option C??

I somehow feel that this is not the right question to practice for GMAT. Am I correct in saying this??


I don't think this is an official LSAT question. Option (A) doesn't make much sense to me and neither does any other option.
Would request a screenshot of the question and solution given.
Current Student
Joined: 31 Jul 2017
Status:He came. He saw. He conquered. -- Going to Business School -- Corruptus in Extremis
Posts: 1734
Own Kudos [?]: 5734 [0]
Given Kudos: 3054
Location: United States (MA)
Concentration: Finance, Economics
Send PM
Newspaper subscriber: Arnot's editorial argues that by making certain [#permalink]
Expert Reply
VeritasKarishma, the screenshot is attached. As the one who uploaded this, I can assure you that this, indeed, an official LSAT question. It is question #14, of section 3, of Prep test 64. I will provide an OE in a different post once I have time to write out a thorough explanation.
Attachments

Veritas.PNG
Veritas.PNG [ 202.68 KiB | Viewed 6697 times ]

Intern
Intern
Joined: 20 Nov 2017
Posts: 8
Own Kudos [?]: 2 [0]
Given Kudos: 91
Send PM
Re: Newspaper subscriber: Arnot's editorial argues that by making certain [#permalink]
can anyone explain why option B is incorrect
Tutor
Joined: 16 Oct 2010
Posts: 14816
Own Kudos [?]: 64880 [0]
Given Kudos: 426
Location: Pune, India
Send PM
Re: Newspaper subscriber: Arnot's editorial argues that by making certain [#permalink]
Expert Reply
nightblade354 wrote:
VeritasKarishma, the screenshot is attached. As the one who uploaded this, I can assure you that this, indeed, an official LSAT question. It is question #14, of section 3, of Prep test 64. I will provide an OE in a different post once I have time to write out a thorough explanation.


Ok. If possible, put up the official explanation too please. I am not convinced of the options so would like to read what the official test maker has to say.
Current Student
Joined: 31 Jul 2017
Status:He came. He saw. He conquered. -- Going to Business School -- Corruptus in Extremis
Posts: 1734
Own Kudos [?]: 5734 [0]
Given Kudos: 3054
Location: United States (MA)
Concentration: Finance, Economics
Send PM
Newspaper subscriber: Arnot's editorial argues that by making certain [#permalink]
Expert Reply
VeritasKarishma, I do not believe I have access to the OE. To be honest, I am skeptical that they even exist in the books.

Here is MGMAT's: https://www.manhattanprep.com/lsat/foru ... t5498.html
Here is Powerscore's: https://forum.powerscore.com/lsat/viewtopic.php?t=1909

And you have mine above. If you have further questions, please let me know.
VP
VP
Joined: 11 Aug 2020
Posts: 1263
Own Kudos [?]: 201 [0]
Given Kudos: 332
Send PM
Re: Newspaper subscriber: Arnot's editorial argues that by making certain [#permalink]
C is the 'best' choice...A is categorically wrong. The author does NOT repudiate the editorial on the grounds that the argument is INADEQUATE, he/she merely expresses skepticism about whether the desired outcome is achievable, reasoning that perhaps the government's interests are not aligned with the goal of achieving greater social good.
Intern
Intern
Joined: 07 Apr 2022
Posts: 4
Own Kudos [?]: 1 [0]
Given Kudos: 47
Location: Norway
Send PM
Re: Newspaper subscriber: Arnot's editorial argues that by making certain [#permalink]
CEdward wrote:
C is the 'best' choice...A is categorically wrong. The author does NOT repudiate the editorial on the grounds that the argument is INADEQUATE, he/she merely expresses skepticism about whether the desired outcome is achievable, reasoning that perhaps the government's interests are not aligned with the goal of achieving greater social good.


C is descriptively wrong. Had answer choice C been "it fails to consider that even if an argument's assumption is false, the conclusion may nonetheless be true," it would be the correct answer.

However, answer choice C says the opposite of what I described.

A is categorically correct. Stating that a conclusion is false simply because such a conclusion was inadequately constructed is a flaw. Think of a bad lawyer representing an innocent person wrongly accused of committing a crime. The person is innocent regardless of the bad arguments that the lawyer might use on trial.

Also, the newspaper subscriber concludes that the argument is inadequate by saying that "clearly this conclusion is false." The subscriber's premise is that Arnot's assumption is dubious.
Intern
Intern
Joined: 07 Apr 2022
Posts: 4
Own Kudos [?]: 1 [0]
Given Kudos: 47
Location: Norway
Send PM
Re: Newspaper subscriber: Arnot's editorial argues that by making certain [#permalink]
SnigdhaM wrote:
can anyone explain why option B is incorrect


"it treats a change that is required for virtual elimination of society's most vexing social ills as a change that will guarantee the virtual elimination of those ills"

I think the author is trying to trick you into thinking that the subscriber confused necessity with requirement.

However, I fail to see where Arnot's editorial argued for a requirement. Moreover, even if it did, I don't see the subscriber confusing anything. The subscriber simply concludes that the editorial's claim is false because the subscriber is suspicious of the assumption made by Arnot's editorial.
GMAT Club Bot
Re: Newspaper subscriber: Arnot's editorial argues that by making certain [#permalink]
Moderators:
GMAT Club Verbal Expert
6917 posts
GMAT Club Verbal Expert
238 posts
CR Forum Moderator
832 posts

Powered by phpBB © phpBB Group | Emoji artwork provided by EmojiOne